Maths > Complex Numbers > 5.0 Representation of complex number

  Complex Numbers
    1.0 Definition
    2.0 Algebraic operations
    3.0 Conjugate of complex number
    4.0 Properties of conjugate
    5.0 Representation of complex number
    6.0 Euler's formulae
    7.0 Properties of Argument
    8.0 De Moivre's Theorem
    9.0 Square root of a complex number
    10.0 The ${n^{th}}$ root of unity
    11.0 Cube roots of unity
    12.0 Rotation
    13.0 Geometrical properties
    14.0 Locus
    15.0 Ptolemy's Theorem

5.3 Properties of modulus
1. $\left| z \right| = 0 \Rightarrow z = 0$



2. $\left| z \right| = \left| {\overline z } \right| = \left| { - \overline z } \right| = \left| { - \overline z } \right|$



3. $z\overline z = {\left| z \right|^2}$



4. $\left| {{z_1}{z_2}} \right| = \left| {{z_1}} \right|\left| {{z_2}} \right|$



5. $\left| {\frac{{{z_1}}}{{{z_2}}}} \right| = \frac{{\left| {{z_1}} \right|}}{{\left| {{z_2}} \right|}}$



6. $\left| {{z_1} \pm {z_2}} \right| \leqslant \left| {{z_1}} \right| + \left| {{z_2}} \right|$



7. $\left| {{z_1} \pm {z_2}} \right| \geqslant \left| {\left| {{z_1}} \right| - \left| {{z_2}} \right|} \right|$



8. $\left| {{z^n}} \right| = {\left| z \right|^n}$



9. $\left| {\left| {{z_1}} \right| - \left| {{z_2}} \right|} \right| \leqslant \left| {{z_1} \pm {z_2}} \right| \leqslant \left| {{z_1}} \right| + \left| {{z_2}} \right|$

Proof: Using triangle inequality in $\Delta OAC$, as shown in figure
$$\begin{equation} \begin{aligned} OC \leqslant OA + AC \\ OA \leqslant AC + OC \\ AC \leqslant OA + OC \\\end{aligned} \end{equation} $$ Using these inequalities, we have $\left| {\left| {{z_1}} \right| - \left| {{z_2}} \right|} \right| \leqslant \left| {{z_1} + {z_2}} \right| \leqslant \left| {{z_1}} \right| + \left| {{z_2}} \right|$
and
Similarly from $\Delta OAB$, we have $\left| {\left| {{z_1}} \right| - \left| {{z_2}} \right|} \right| \leqslant \left| {{z_1} - {z_2}} \right| \leqslant \left| {{z_1}} \right| + \left| {{z_2}} \right|$


10. ${\left| {{z_1} \pm {z_2}} \right|^2} = \left| {{z_1} \pm {z_2}} \right|\overline {\left| {{z_1} \pm {z_2}} \right|} = \left| {{z_1} \pm {z_2}} \right|\left| {\overline {{z_1}} \pm \overline {{z_2}} } \right|$
${\text{ = }}{\left| {{z_1}} \right|^2} + {\left| {{z_2}} \right|^2} \pm \left( {{z_1}\overline {{z_2}} + \overline {{z_1}} {z_2}} \right)$


Question 4. If $\left| {{z_1}} \right| = 1,{\text{ }}\left| {{z_2}} \right| = 1,{\text{ }}\left| {{z_3}} \right| = 1,{\text{ }}\left| {{z_1} + {z_2} + {z_3}} \right| = k$, find the value of $$\left| {\frac{1}{{{z_1}}} + \frac{1}{{{z_2}}} + \frac{1}{{{z_3}}}} \right|$$

Solution: As we know that, $z\overline z = {\left| z \right|^2}$, therefore,
$$\begin{equation} \begin{aligned} {z_1}\overline {{z_1}} = 1,{\text{ }}{z_2}\overline {{z_2}} = 1,{\text{ }}{z_3}\overline {{z_3}} = 1 \\ \Rightarrow \frac{1}{{{z_1}}} = \overline {{z_1}} ,{\text{ }}\frac{1}{{{z_2}}} = \overline {{z_2}} ,{\text{ }}\frac{1}{{{z_3}}} = \overline {{z_3}} \\ \therefore \left| {\frac{1}{{{z_1}}} + \frac{1}{{{z_2}}} + \frac{1}{{{z_3}}}} \right| = \left| {\overline {{z_1}} + \overline {{z_2}} + \overline {{z_3}} } \right| = \left| {\overline {{z_1} + {z_2} + {z_3}} } \right| \\ \because \left| z \right| = \overline {\left| z \right|} \\ \Rightarrow \left| {\frac{1}{{{z_1}}} + \frac{1}{{{z_2}}} + \frac{1}{{{z_3}}}} \right| = \left| {{z_1} + {z_2} + {z_3}} \right| = k \\\end{aligned} \end{equation} $$


Question 5. If $\left| {\frac{{{z_1} - 2{z_2}}}{{2 - {z_1}\overline {{z_2}} }}} \right| = 1$ in which $\left| {{z_2}} \right| \ne 1$. Find $\left| {{z_1}} \right|$.

Solution: $$\begin{equation} \begin{aligned} \left| {\frac{{{z_1} - 2{z_2}}}{{2 - {z_1}\overline {{z_2}} }}} \right| = 1 \\ \left| {{z_1} - 2{z_2}} \right| = \left| {2 - {z_1}\overline {{z_2}} } \right| \\\end{aligned} \end{equation} $$
Squaring both sides, we get
$$\begin{equation} \begin{aligned} {\left| {{z_1} - 2{z_2}} \right|^2} = {\left| {2 - {z_1}\overline {{z_2}} } \right|^2} \\ \left| {{z_1} - 2{z_2}} \right|\overline {\left| {{z_1} - 2{z_2}} \right|} = \left| {2 - {z_1}\overline {{z_2}} } \right|\overline {\left| {2 - {z_1}\overline {{z_2}} } \right|} \\ \left( {{z_1} - 2{z_2}} \right)\left( {\overline {{z_1}} - 2\overline {{z_2}} } \right) = \left( {2 - {z_1}\overline {{z_2}} } \right)\left( {2 - \overline {{z_1}} {z_2}} \right) \\ {\left| {{z_1}} \right|^2} - 2{z_1}\overline {{z_2}} - 2{z_1}\overline {{z_2}} + 4{\left| {{z_2}} \right|^2} = 4 - 2\overline {{z_1}} {z_2} - 2\overline {{z_1}} {z_2} + {\left| {{z_1}} \right|^2}{\left| {{z_2}} \right|^2} \\ \left( {{{\left| {{z_1}} \right|}^2} - 4} \right) - {\left| {{z_2}} \right|^2}\left( {{{\left| {{z_1}} \right|}^2} - 4} \right) = 0 \\ \left( {{{\left| {{z_1}} \right|}^2} - 4} \right)\left( {1 - {{\left| {{z_2}} \right|}^2}} \right) = 0 \\ {\left| {{z_1}} \right|^2} = 4 \Rightarrow {z_1} = 2 \\ {\left| {{z_2}} \right|^2} = 1 \Rightarrow {z_2} = 1,{\text{ not possible}}{\text{.}} \\\end{aligned} \end{equation} $$


Question 6. Find the greatest and least value of the modulus of complex number $z$ satisfying the equation $$\left| {z - \frac{4}{z}} \right| = 2$$

Solution: From figure, we have
$$\begin{equation} \begin{aligned} \left| {OP - OQ} \right| \leqslant QP \\ \Rightarrow \left| {\left| z \right| - \left| {\frac{4}{z}} \right|} \right| \leqslant \left| {z - \frac{4}{z}} \right| = 2 \\ \Rightarrow - 2 \leqslant \left| z \right| - \left| {\frac{4}{z}} \right| \leqslant 2 \\ \Rightarrow {\left| z \right|^2} + 2\left| z \right| - 4 \geqslant 0{\text{ or }}{\left| z \right|^2} - 2\left| z \right| - 4 \leqslant 0 \\ {\left( {\left| z \right| + 1} \right)^2} - 5 \geqslant 0{\text{ or }}{\left( {\left| z \right| - 1} \right)^2} - 5 \leqslant 0 \\ \left( {\left| z \right| + 1 + \sqrt 5 } \right)\left( {\left| z \right| + 1 - \sqrt 5 } \right) \geqslant 0{\text{ or }}\left( {\left| z \right| - 1 + \sqrt 5 } \right)\left( {\left| z \right| - 1 - \sqrt 5 } \right) \leqslant 0 \\ \Rightarrow \sqrt 5 - 1 \leqslant \left| z \right| \leqslant \sqrt 5 + 1 \\\end{aligned} \end{equation} $$
Therefore, greatest value of ${\left| z \right|}$ is $\sqrt 5 + 1$ and least value of ${\left| z \right|}$ is $\sqrt 5 - 1$.
Improve your JEE MAINS score
10 Mock Test
Increase JEE score
by 20 marks
Detailed Explanation results in better understanding
Exclusively for
JEE MAINS and ADVANCED
9 out of 10 got
selected in JEE MAINS
Lets start preparing
DIFFICULTY IN UNDERSTANDING CONCEPTS?
TAKE HELP FROM THINKMERIT DETAILED EXPLANATION..!!!
9 OUT OF 10 STUDENTS UNDERSTOOD